LSAT and Law School Admissions Forum

Get expert LSAT preparation and law school admissions advice from PowerScore Test Preparation.

 jbrown1104
  • Posts: 23
  • Joined: Jun 15, 2018
|
#46784
Hello PS!

I am still struggling to see how answer choice (E) justifies the conclusion. While I was able to isolate the conclusion within the stimulus and the supporting premise, I still do not understand how (E) "the total amount of money spent on treating disease X slowly declined during the past decade" abides by the Justify Formula.

Perhaps it was my rephrase that threw me off. Mine was "over the past decade there was an increase in spending on nonstandard treatment".

Thanks!
User avatar
 Jonathan Evans
PowerScore Staff
  • PowerScore Staff
  • Posts: 726
  • Joined: Jun 09, 2016
|
#46791
Hi, JBrown,

Thanks for the question!

Yes, your prephrase might have been part of the issue here. Let's do a quick diagram of this problem for our discussion:
  1. Decreasing percentage of Disease X treatment money spent on Standard Treatments.
  2. Increasing percentage of Disease X treatment money spent on Non-Standard Treatments.
  3. Standard Treatments are Effective.
  4. Non-Standard Treatments are not Effective.
We may assume that treatments are either Standard or Non-Standard.

Unknowns include:
  • Total amount of money spent on Disease X treatments.
  • Amount spent on Standard Treatments.
  • Amount spent on Non-Standard Treatments.
The conclusion is:
  • Less money is spent on Effective treatments.
This conclusion deals with one of our unknowns: "Amount spent on Standard Treatments."

We do not know the actual amount spent on Standard Treatments. However, we do know the percentage spent on these Standard Treatments has been decreasing.

Let's consider your prephrase:
"over the past decade there was an increase in spending on nonstandard treatment"
Even if we were to know that the amount spent on nonstandard treatments increased, we would still not know that the actual amount spent on Standard, Effective treatments decreased. It could be possible that both amounts increased, just with Non-Standard increasing more quickly, or with the amount spent on Standard Treatments remaining the same. Let me illustrate:
  • Starting amounts: $50 for ST. $50 for NST.
    New amounts: $50 for ST. $100 for NST.
Even though the NST amount has gone up, the ST amount has not gone down. However, the percentage spent on ST has declined from 50% to 33%, which agrees with the premises.

To justify the conclusion that the actual amount on ST went down, we need some additional information that we can add to the premises that guarantees that the conclusion is valid (this is the Justify Equation™).

Answer choice (E) does this. Let's illustrate:
  • Starting amounts: $50 for ST. $50 for NST.
    New TOTAL amount: $80.
If the new total amount went down from $100 to $80 and if the percentage spent on ST has also declined, then it is a mathematical certainty that the amount spent on ST has gone down. It must be less than $50.

Does this make sense? Great question.
 jbrown1104
  • Posts: 23
  • Joined: Jun 15, 2018
|
#46887
Thank you Johnathan I think I get it now.

So, even though the conclusion states that less money is spent on standard treatments, we cannot assume that MORE money is spent on nonstandard treatments. AND because within the stimulus there is no start amount for either treatment that the author gives us, the ONLY assumption that can be made is that money spent for the treatment of disease X as a WHOLE has been reduced, because the author does not provide us with any information to assume that one form of treatments increased while another decreased.

Is this correct?

Thank you!
 Adam Tyson
PowerScore Staff
  • PowerScore Staff
  • Posts: 5153
  • Joined: Apr 14, 2011
|
#47025
It's not that we have to assume that less money is being spent, JB, but that IF we assume that to be true, THEN the conclusion of the argument must be true. A Justify the Conclusion answer isn't necessary for the argument to be valid, but it is sufficient to prove the conclusion is true.

Other than that, you're spot on! Nice job!
 rcthomas23
  • Posts: 4
  • Joined: Nov 23, 2021
|
#98193
Hello,

I had trouble trying to solve this Justify question mechanistically. Would that be the correct approach for this question?

Thank you!
 Robert Carroll
PowerScore Staff
  • PowerScore Staff
  • Posts: 1787
  • Joined: Dec 06, 2013
|
#98203
rcthomas23,

You could solve this mechanistically. The conclusion talks about an amount of money, whereas the premises are entirely about percents. So you'd need to show that the decreasing percentage spent on standard methods of treatment actually represents less money. That would work if less money overall were spent on all treatments - a decreasing percentage of a decreasing total amount is definitely less money.

Robert Carroll
User avatar
 cd1010
  • Posts: 34
  • Joined: Jul 12, 2022
|
#105392
I got this, but I wanted to clarify the explanation for B. Administrator response is:
Answer choice (B): Like answer choice (A) above, this choice provides an irrelevant consideration. The price of one type of treatment vs. the other tells us nothing about the comparison between dollars spent in treatment today vs. dollars spent ten years ago.
But B does compare cost today of nonstandard treatments vs cost 10 years ago of non-standard treatments. I eliminated this because this comparison of costs just for nonstandard treatment is still not sufficient to say anything about money spent on standard treatment.
 Robert Carroll
PowerScore Staff
  • PowerScore Staff
  • Posts: 1787
  • Joined: Dec 06, 2013
|
#105395
cd1010,

I'm going to come to the defense of the explanation. Information about the comparison between standard vs nonstandard isn't relevant because the conclusion is about the total. The extra information in answer choice (B) doesn't even tell me that the total spent on nonstandard treatments is higher, and even if that were the case, I'd still know nothing about standard treatments. Answer choice (B) is essentially telling you that nonstandard treatment are more expensive per treatment. That's the wrong direction to go - we already weren't happy with the comparison of the parts (standard vs nonstandard), and extra information about a "part of a part" (the per-treatment cost of the nonstandard segment of treatments) is giving no info about total expenses for all treatments, which is what the conclusion wants.

I'm also going to say that your reason for eliminating answer choice (B) is also a perfectly good reason to say it's wrong!

Robert Carroll

Get the most out of your LSAT Prep Plus subscription.

Analyze and track your performance with our Testing and Analytics Package.